universidade federal do rio de janeiro ...dcc.ufrj.br/~collier/cursosgrad/calcnum/gabaritos.pdfcomo...

35

Upload: phunglien

Post on 06-May-2018

218 views

Category:

Documents


1 download

TRANSCRIPT

UNIVERSIDADE FEDERAL DO RIO DE JANEIRO

Departamento de Ciência da Computação�UFRJ

Cálculo Numérico

S. C. Coutinho

Provas e gabaritos

Lembre-se: Nas provas não são aceitas respostas sem justi�cativa. Você deve saberexplicar tudo o que �zer.

DCC-UFRJ�Cálculo numérico

Primeira Prova�Turma EC2�2015/2

Questão 1 (4 pontos)Considere as funções f1(x) = ex/4 e f2(x) = 1, 982/x.

(a) Invente uma função g(x), diferente da que é dada pelo método de Newton, cujoponto �xo é o ponto de interseção dos grá�cos de y = f1(x) com y = f2(x).

(b) Veri�que que a iteração dada por xn+1 = g(xn) é convergente no intervalo [0, 2].

(c) Determine uma aproximação numérica, correta até a segunda casa decimal, doponto �xo de g(x), partindo do ponto x0 = 1, 3.

(d) Determine uma aproximação numérica, correta até a segunda casa decimal, doponto de interseção de y = f1(x) com y = f2(x), usando o método de Newton,com ponto de partida x0 = 1, 3.

Solução:

O ponto de interseção é dado por f1(x) = f2(x); isto é, por ex/4 = 1, 982/x, quepodemos reescrever na forma x = 1, 982/ex/4. Isto sugere tomar

g(x) = 1, 982e−x/4.

Para mostrar que a iteração converge em [0, 2], usamos o teorema do valor médiopara escrever

|xn+1 − ξ| = |g(xn)− g(ξ)| = |g′(c)||xn − ξ|,

em que ξ é o ponto �xo de g e c está entre xn e ξ. Mas

g′(x) = −1, 982

4e−x/4.

Como ex/4 é uma função crescente, sua inversa e−x/4 = 1/ex/4 é decrescente. Logo,

|g′(x)| = 1, 982

4ex/4<

1, 982

4≤ 0.4995 < 1,

para todo x ≥ 0. Iterando a função g a partir de x0 = 1, 3, obtemos os seguintesvalores para xn e o erro en:

n xn en1 1, 432 · 100 1, 321 · 10−12 1, 386 · 100 4, 651 · 10−23 1, 402 · 100 1, 617 · 10−24 1, 396 · 100 5, 676 · 10−3

Portanto, a aproximação desejada é x4 = 1.396. Finalmente, precisamos acharum zero de ex/4 = 1, 982/x usando o método de Newton. Para facilitar as contas,vou rearrumar a equação na forma xex/4 − 1, 982 = 0, de modo que o problema

Page 2

consiste em calcular um zero da função h(x) = xex/4 − 1, 982 pelo método deNewton. Como

h′(x) = ex/4 +xex/4

4=x+ 4

4ex/4

a iteração do método de Newton-Raphson é dada por

xn+1 = xn − 4xne

x/4 − 1, 982

(xn + 4)exn/4=x2ne

xn/4 + 7, 932

(xn + 4)exn/4.

Calculando a iteração pedida, temos

n xn en1 1.4002 0.10022 1.3980 0.0021

Portanto, a aproximação desejada é 1.3980.

Questão 2 (4 pontos)Considere o sistema linear

x+ 4y + z = 7

3x+ y − z = 3

−5x+ 13y − 22z = 48.

(a) Calcule a decomposição PLU da matriz do sistema.

(b) Calcule a solução exata do sistema.

(c) Rearrume o sistema de modo que o método de Jacobi seja convergente e calculeduas iterações por este método, partindo de v(0) = [0, 0, 0]t

(d) Calcule os erros absoluto e relativo cometidos, no cálculo feito em (c), para acordenada y da solução do sistema, arredondando para três casas decimais.

Solução:

A matriz do sistema é 1 4 13 1 −1−5 13 −22

Vamos aplicar eliminação gaussiana a esta matriz. Como a posição 1, 1 é nãonula, não há necessidade de trocar linhas de posição. Ao �nal da eliminação como pivô na posição 1, 1, obtemos:

U =

1 4 10 −11 −40 33 −17

e L =

1 0 03 1 0−5 0 1

Page 3

Mas uma vez não há necessidade de trocar linhas de lugar, porque a posição 2, 2não é nula. Ao �nal desta etapa, obteremos

U =

1 4 10 −11 −40 0 −29

e L =

1 0 03 1 0−5 −3 1

;

além de P , que será igual à matriz identidade 3×3. Por outro lado, multiplicando 1 4 13 1 −1−5 13 −22

xyz

=

7348

à esquerda por L−1, obtemos1 4 1

0 −11 −40 0 −29

xyz

=

1 0 0−3 1 05 3 1

7348

=

7−1829

,donde x = 0, y = 2 e z = −1. Passando à letra (c), trocamos a primeira equaçãoo sistema com a segunda, para obter a matriz

A =

3 1 −11 4 1−5 13 −22

cuja diagonal é estritamente dominante. Com isto o método de Jacobi convergepara esta matriz. Não esqueça que também preciamos trocar as posições das duasprimeiras entradas no vetor de constante, que passa a ser

b =

3748

,Como 3 1 −1

1 4 1−5 13 −22

=

3 0 00 4 00 0 −22

+

0 1 −11 0 1−5 13 0

,o sistema pode ser reescrito na forma3 0 0

0 4 00 0 −22

xyz

= −

0 1 −11 0 1−5 13 0

xyz

+

3748

,da qual extraímos a iteração do método de Jacobi:3 0 0

0 4 00 0 −22

xn+1

yn+1

zn+1

= −

0 1 −11 0 1−5 13 0

xnynzn

+

3748

.

Page 4

Aplicando duas vezes esta iteração com v0 = [0, 0, 0]t, obtemos

v1 =

3748

. e v2 =

−0.310606060606062.045454545454545−1.375

.Com isso, o erro absoluto cometido no cálculo da coordenada y é∣∣∣∣22931056

− 2

∣∣∣∣ = 181

1056= 0.17140151515151 ≈ 0.171

e o erro relativo correspondente é

1811056

2=

181

2112= 0.085700757575757 ≈ 0.086.

Questão 3 (3 pontos)Dê exemplo de:

(a) uma função f : R → R que tem um único zero, mas para a qual o método debisseção não funciona;

(b) uma matriz A, de tamanho 2 × 2 e com 1 nas duas posições da diagonal, demodo que a matriz R correspondente à iteração xn+1 = Rxn + c do método de

Gauss-Seidel tem raio espectral maior que 1;

(c) um polinômio de grau dois para o qual o método de Newton alterna entre osvalores 1 e 2.

Solução:

Um exemplo para (a) é f(x) = x2, porque todo o grá�co está de um lado só doeixo x, de modo que não podemos aplicar o teorema do valor intermediário. Para(b), vou considerar a matriz

A =

[1 bc 1

]=

[1 0c 1

]+

[0 b0 0

]e calcular

R = −(D + L)−1U = −[1 0c 1

]−1 [0 b0 0

].

Como [1 0c 1

]−1=

[1 0−c 1

]de modo que

R = −(D + L)−1U = −[1 0−c 1

] [0 b0 0

]=

[0 −b0 cb

]

Page 5

Para que R tenha raio espectral maior que 1 é necessário que |bc| > 1. Finalmente,para resolver (c), suporemos que f(x) = x2 + ax + b. Calculando a iteração dométodo de Newton-Raphson para este polinômio, obtemos

g(x) = x− x2 + ax+ b

2x+ a=x2 − b2x+ a

.

Queremos que

g(1) =1− b2 + a

= 2 e que g(2) =4− b4 + a

= 1;

que corresponde ao sistema linear

2a+ b = −3a+ b = 0.

Resolvendo o sistema, obtemos a = −3 e b = 3, de modo que o polinômio desejadoé x2 − 3x+ 3.

Page 6

DCC-UFRJ�Cálculo numérico

Segunda Prova�Turma EC2�2015/2

Questão 1 (3 pontos)A tabela abaixo foi obtida como resultado de um experimento relativo à variação datemperatura T (em graus Celsius) com a posição x (em centímetros):

T 22 43 84 210 320x 0.1 0.2 0.4 0.8 0.9

(a) Use interpolação entre os pontos de posição 0.1, 0.2 e 0.4 para calcular a tem-peratura na posição 0.3 com arredondamento para três casas decimais.

(b) Determine a curva da forma T = aebx que melhor se ajusta aos dados da tabelae use a fórmula assim obtida para calcular T (0.3) com três casas decimais.

Solução:

O polinômio interpolador é

P = 22(x− 0.2)(x− 0.4)

(0.1− 0.2)(0.1− 0.4)+43

(x− 0.1)(x− 0.4)

(0.2− 0.1)(0.2− 0.4)+84

(x− 0.1)(x− 0.2)

(0.4− 0.1)(0.4− 0.1).

Substituindo x = 0.3, obtemos P (0.3) = 63.667. Para achar a curva exponencialT = aebx que melhor se ajusta a estes dados aplicamos logaritmo natural a estaequação obtendo

ln(T ) = ln(a) + bx.

Escrevendo α = ln(a) a equação toma a forma

ln(T ) = α + bx.

Para poder montar o sistema, precisamos dos logaritmos dos valores de T dadosna tabela:

T 22 43 84 210 320ln(T ) 3.091 3.761 4.431 5.347 5.768x 0.1 0.2 0.4 0.8 0.9

A matriz de Vandermonde correspondente é

V =

1 0.11 0.21 0.41 0.81 0.9

de modo que a equação normal é dada por

V tV

[αb

]= V tb

Page 7

em que

b =

224384210320

Como

V tV =

[5 2.42.4 1.66

]e V tb =

[22.39812.303

]obtemos, ao resolver o sistema, que

α = 3.014 e b = 3.054.

Levando em conta que a = eα = 20.37, a relação entre T e x que melhor se adaptaaos dados é T = 20.37 exp(3.054x). A aproximação para T (0.3) resultante destaexpressão é 50.92.

Questão 2 (3 pontos)A área do círculo x2 + y2 = 1 é igual a π.

(a) Determine uma aproximação para a área limitada por este círculo no primeiroquadrante usando o método de Simpson com h = 0.25 e determine uma estima-tiva para π a partir disto. Expresse o resultado com três casas decimais.

(b) Seja f(x) =√1− x2. Sabendo-se que f ′′(0) = −1, que f ′′(

√2/2) = −

√2/2 e

que f ′′′(x) não se anula no intervalo aberto (0, 1), determine h de modo que aintegração pela regra do trapézio produza o valor de π correto até a segundacasa decimal.

Solução:

Se f(x) =√1− x2 então, pelo método de Simpson,∫ 1

0

f(x)dx =h

3(f(x0) + f(x4) + 2f(x2) + 4(f(x1) + f(x3)))

Tabelando os valores de f(xi) obtemos

i 0 1 2 3 4xi 0.0 0.25 0.5 0.75 1.0f(x) 1.0 0.9683 0.866 0.6614 0.0

Substituindo na fórmula e efetuando os cálculos∫ 1

0

f(x)dx =0.25

3(1.0 + 0.0 + 2 · 0.866 + 4(0.9683 + 0.6614)) = 0.7709.

Page 8

Arredondando para 3 casas decimais, obtemos 0.771, de modo que o valor de πcorrespondente será 4 · 0.771 = 3.084. Para obter π correto até a segunda casadecimal com os dados de (b) precisamos que a integral entre 0 e

√2/2 seja igual

a 3.14/4 = 0.785 quando calculada com 4 decimais corretas. Pela fórmula do erropara o método do trapézio devemos ter, portanto, que

10−3 >

∣∣∣∣∣(0−√2)h2f ′′(ξ)

12

∣∣∣∣∣ ,para algum ξ ∈ (0,

√2/2). Como f ′′′(x) não se anula em (0, 1), os valores dados

para f ′′(x) mostram que a |f ′′(x)| é crescente em (0,√2/2). Logo, considerando

o intervalo de integração como sendo [0,√2/2], temos que

10−3 >

∣∣∣∣∣√2h2f ′′(ξ)

12

∣∣∣∣∣ >√2h2 · 112

=

√2h2

12.

Segue-se disto que

h2 <12 · 10−3√

2≈ 0.0085;

donde teria que ser menor que 0.0921.

� Note que não é possível usar o intervalo [0, 1] no cálculo do erro porque a função

f ′′(x) = − x√1− x2

não é limitada neste intervalo.

Questão 3 (3 pontos)Considere o problema de valor inicial

y′ − y2 cos(x) = 0 e y(0) = 1.

(a) Descreva a recorrência do método de Euler modi�cado no caso especí�co doproblema de valor inicial acima.

(b) Calcule o valor de y(1) usando o método de Euler modi�cado com passo 0.5.

Sua resposta deve incluir todos os valores intermediários das variáveis calculados aolongo da execução do algoritmo.

Solução:

Page 9

A iteração é dada por

y(0) = 1

y∗n+1 = yn + hy2n cos(xn)

yn+1 = yn +h

2

(y2n cos(xn) + (y∗n+1)

2 cos(xn+1))

Aplicando-a com xn = n · 0.5, obtemos os dados tabelados abaixo:

n 0 1 2xn 0 0.5 1y∗n × 1.5 3.08yn 1 1.74 3.69

Portanto, o valor desejado é y(1) = 3.69.

Questão 4 (2 pontos)Considere o problema de valores de contorno

y′′ = 3y′ + y + x2, y(0) = −20 e y(3) = −11.

Calcule y(1) usando o método de diferenças �nitas com h = 1.

Solução:

Substituindo as aproximações

y′(xn) ≈yn−1 + yn+1

2he y′′(xn) ≈

yn+1 − 2yn + yn−1h2

na equação e levando em conta que h = 1, obtemos

yn+1 − 2yn + yn−1 =3(yn+1 + yn−1)

2+ yn + x2n,

donde, quando n = 1,

2(y2 − 2y1 + y0) = 3(y2 + y0) + 2y1 + 2 · 12

e, quando n = 2,

2(y3 − 2y2 + y1) = 3(y3 + y1) + 2y2 + 2 · 22.

Levando em conta que y(0) = −20 e y(2) = −11, obtemos

−6y1 − y2 = 102

5y1 − 6y2 = −3.

Resolvendo o sistema y1 = −15 e y2 = −12. Portanto, y(1) ≈ −15.

Page 10

DCC-UFRJ�Cálculo numérico

Primeira Prova�Ciência da Computação�2016/2

Questão 1 (4 pontos)Considere o sistema linear AX = b, em que

A =

9 3 6−1 5 2−3 −1 12

e b =

123

(a) Calcule a decomposição PLU da matriz do sistema, usando pivoteamento parcial.

(b) Calcule as matrizes R e c tais que x = Rx + c é a iteração obtida aplicando-seo método de Gauss a este sistema.

(c) Calcule o autovalor dominante de R com erro menor que 10−1 usando o métododa potência a partir de u0 = [1, 1, 1]t/

√3.

(d) O que o resultado obtido em (c) nos diz sobre a convergência da iteração xn+1 =Rxn + c?

Solução:

Aplicando eliminação gaussiana, temos 9 3 6 | 1 | 1 0 0−1 5 2 | 2 | 0 1 0−3 −1 12 | 3 | 0 0 1

→9 3 6 | 1 | 1 0 00 16/3 8/3 | 2 | −1/9 1 00 0 14 | 3 | −1/3 0 1

Portanto, P é a matriz identidade,

U =

9 3 60 16/3 8/30 0 14

e L =

1 0 0−1/9 1 0−1/3 0 1

(b) Decompondo A na forma

A =

9 3 6−1 5 2−3 −1 12

=

9 0 0−1 5 0−3 −1 0

+

0 3 60 0 20 0 0

temos que

R = −

9 0 0−1 5 0−3 −1 12

−1 ·0 3 60 0 20 0 0

e c =

9 0 0−1 5 0−3 −1 12

−1 ·123

Como 9 0 0

−1 5 0−3 −1 12

−1 = 1

90 0

145

15

04

135160

112

,

Page 11

então,

R = −

0 13

23

0 115

815

0 445

1990

e c =

2.6671.7330.8111

.� Várias pessoas �zeram Jacobi, em vez de Gauss-Seidel.

(c) Ao �nal do primeiro laço temos

w1 = R · u0 = −

0 13

23

0 115

815

0 445

1990

0.5770.5770.577

= −

0.57740.34640.1732

normalizando w1 e calculando a aproximação do autovalor correspondente, obte-mos

u1 = −

0.83040.49830.2491

e λ1 = ut1Ru1 = −0.3827.

Comoλ0 = ut0Ru0 = −0.6334.

Como o erro será

|λ0 − λ1| = | − 0.6334 + 0.3827| = 0.2507

é maior que 0.1 precisamos executar mais um laço. Desta vez

w2 =

0.33220.16610.09688

donde u2 =

0.86550.43270.2524

e λ2 = −0.3644.

Como|λ1 − λ2| = | − 0.3644 + 0.3827| = 0.0183 < 0.1,

o processo para. Logo a aproximação desejada para o autovalor dominante é−0.3644.

� Algumas pessoas iteraram A, em vez de R.

(d) Como o maior autovalor em módulo é 0.3644, o raio espectral de R tem queser menor que 1. Logo, a iteração do método de Gauss-Seidel converge para asolução do sistema.

� A pergunta diz respeito à convergência de Gauss-Seidel e não à convergência do

método da potência.

Questão 2 (6 pontos)Considere a função f(x) = x cos(x)− x2 − 8x− 1 com domínio no intervalo [−1, 0].

Page 12

(a) Determine uma função g(x) cujo ponto �xo é um zero de f(x) e prove que aiteração xn+1 = g(xn) converge no intervalo [−1, 0].

(b) Use esta iteração com x0 = 0 para achar o zero de f(x) com erro inferior a 10−2.

(c) Calcule o polinômio interpolador pelos pontos (f(xi), xi), em que x0 = −1,x1 = −0.5 e x2 = 0.

(d) Calcule o zero de f(x) (arredondado para duas casas decimais) usando o polinô-mio interpolador e determine o erro absoluto que seria cometido se achássemoso zero por este método.

(e) Calcule o polinômio linear que melhor se ajusta aos pontos de (c) usando ométodo dos mínimos quadrados.

O item (a) desta questão vale 2 pontos.

Solução:

f(x) = 0 nos sugere escrever

8x = x cos(x)− x2 − 1

donde

g(x) =x cos(x)− x2 − 1

8.

Para mostrar que esta iteração converge, precisamos calcular a derivada de g(x):

g′(x) =−x sen (x) + cos (x)− 2 x

8.

Como |x|, | cos(x)| e sen(x)| são todos menores ou iguais a 1, temos pela desi-gualdade triangular que

|g′(x)| =∣∣∣∣−x sen(x) + cos(x)− 2x

8

∣∣∣∣ ≤ |x| · | sen(x)|+ | cos(x)|+ 2|x|8

≤ 4

8=

1

2,

para todo x ∈ [−1, 0]. Portanto, a iteração dada por xn+1 = g(xn) para a funçãoxn escolhida realmente converge no intervalo [−1, 0]. Iterando a partir de x0 = 0,temos

i xi g(xi) erro0 0.0 −0.125 0.1251 −0.125 −0.1425 0.01752 −0.1425 −0.1452 0.0027

Logo, a aproximação desejada para o zero de f(x) no intervalo [−1, 0] é −0.1452.(c) e (d) Tabelando os pontos, obtemos

f(xi) −0.3175 −0.2111 −0.125xi −1.0 −0.5 0.0

de modo que o polinômio interpolador, calculado pelo método de Lagrange é

p(x) = −0.0012 x2 − 0.1494 x− 0.1482.

Page 13

O zero de f(x) calculado a partir do polinômio interpolador é p(0) = −0.1482 ≈−0.15 e o erro absoluto, quando calculamos o zero de f(x) desta maneira é

|0.15− 0.14| = 0.01.

� Muita gente errou as questões (c) e (e) porque interpolou os pontos (xi, f(xi)) emvez de (f(xi), xi), como foi pedido.

(e) A matriz de Vandermonde é

V =

1.0 −9.4591.0 −4.8111.0 −1.0

ao passo que b =

[−1 −1

2

0

]

donde

vtV =

[3.0 −15.27−15.27 113.6

]e c =

[−1.511.87

].

Logo, a equação normal é[3.0 −15.27−15.27 113.6

] [ab

]=

[−1.5−6.615

],

cujas soluções sãoa = −0.155 e b = −0.151.

Donde a reta desejada é y = −0.151x− 0.155.

Page 14

DCC-UFRJ�Cálculo numérico

Segunda Prova�Ciência da Computação�2016/2

Questão 1 (2 pontos)Use o método de Newton para calcular o máximo da função f(x) = x(3 − ex/4) nointervalo [2.0, 2.5] com tolerância inferior a 10−2. Você deve veri�car que o ponto queobteve é, de fato, um máximo de f .

Solução:

O máximo é um zero da primeira derivada de f , que é igual a

f ′(x) = (3− ex/4) + x(−1

4ex/4) = 3− (4 + x)

4ex/4.

Portanto, devemos aplicar o método de Newton a esta função. Como

f ′′(x) = −1

4ex/4 − (4 + x)

16ex/4 = −(8 + x)

16ex/4,

a iteração do método de Newton será xk+1 = g(xk), com

g(x) = x− 412− (4 + x)ex/4

−(8 + x)ex/4=

48 + (x2 + 4x− 16)ex/4

(8 + x)ex/4.

Iterando a partir de x = 2.0,

x1 = g(2.0) = 2.511

x2 = g(2.511) = 2.471

x3 = g(2.471) = 2.471

Com isto achamos o ponto crítico x ≈ 2.47, que é, de fato, um máximo, porque

f ′′(2.47) = −2.57.

Questão 2 (3 pontos)Seja In o valor aproximado da integral∫ 1

0

1

x+ 1dx

calculado usando a regra do trapézio com [0, 1] subdividido em n partes iguais.

(a) Prove que esta integral é igual a ln(2).

(b) Calcule I4. A diferença I4 − ln(2) é positiva ou negativa?

Page 15

(c) Explique porque, qualquer que seja n, a diferença In−ln(2) terá sempre o mesmosinal que I4 − ln(2).

Solução:

Fazendo a substituição u = x+ 1 a integral se torna∫ 2

1

1

udu = ln(x)|u=2

u=0 = ln(2)− ln(1) = ln(2).

Para calcular I4, devemos tomar h = 1/4, de modo que, pela regra do trapézio

I4 =1

8

(1 + 2

4

5+ 2

2

3+ 2

4

7+

1

2

)=

1171

1680= 0.697.

Logo, a diferença é

I4 − ln(2) = 0.697− 0.693 = 0.004 > 0.

A diferença será sempre positiva porque a função 1/(x+1) tem concavidade parabaixo. Com isso, qualquer segmento de reta entre dois pontos da curva �ca sempreacima do arco da curva.

Questão 3 (3 pontos)Considere o problema de valor inicial y = t cos(y) e y(0) = 0.

(a) Calcule uma aproximação para y(1) usando o método de Runge-Kutta de se-gunda ordem com h = 0.5.

(b) Use o resultado de (a) para calcular uma aproximação para y(1).

Solução:

Aplicando o método de Runge-Kutta de segunda ordem ao problema dado, obte-mos a iteração

yk+1 = yk + 0.25(tk cos(yk) + (tk + 0.5) cos(yk + 0.5tk cos(yk))).

Como y0 = 0, então y1 = 0.125 e y2 = 0.481. Para (b), usamos a regra da cadeia,para obter de y = t cos(y) que

y =d(t cos(y))

dt= cos(y)− t sen(y)y = cos(y)− t2 sen(y) cos(y)

Portanto,y(1) ≈ cos(y2)− t22 sen(y2) cos(y2) = 0.476.

Page 16

Questão 4 (2 pontos)Considere o problema de valores de contorno

y′′ + 4xy′ = x2, com y(0) = 0 e y(3) = 0.

(a) Determine o sistema linear obtido aplicando-se a este problema o método dasdiferenças �nitas com passo h. Você deve explicitar de que forma as condiçõesde contorno afetam o sistema.

(b) Resolva o sistema para h = 1 e calcule os valores de y(1) e y(2).

Solução:

Substituindo as aproximações

y′(xk) ≈yk+1 − yk−1

2he y′′(xk) ≈

yk+1 − 2yk + yk−1h2

na equação, obtemos

yk+1 − 2yk + yk−1h2

+ 2xkyk+1 − yk−1

h= x2k.

Escrevendon = (3− 0)/h = 3/h

as condições de contorno serão

y0 = yn = 0.

Quando k = 1,y2 − 2y1 + y0

h2+ 2x1

y2 − y0h

= x21;

que, levando em conta y0 = 0 e que x1 = h, torna-se

2h2 + 1

h2y2 −

2

h2y1 = h2.

Por outro lado, quando k = n− 1,

(−2xn−1h+ 1)

h2yn−2 −

2

h2yn−1 = x2n−1.

Portanto, no caso especí�co em que h = 1, o sistema que devemos resolver é

−2y1 + 3y2 = 1

−3y1 − 2y2 = 4,

cujas soluções são

y1 = −14

13e y2 = −

5

13.

Page 17

DCC-UFRJ�Cálculo numérico

Prova Final�Ciência da Computação�2016/2

Questão 1 (3.0 pontos)As seguintes iterações foram propostas como maneiras de calcular a interseção dosgrá�cos das funções sen(x) e f(x) = −2x+ 2:

g1(x) = 2− sen(x)− xg2(x) = (2− sen(x))/2

g3(x) = (− sen(x) + x cos(x) + 2)/(2 + cos(x)).

(a) Explique como cada uma destas iterações foi obtida.

(b) Para quais destas iterações podemos garantir a convergência a partir de x = 1?

(c) Qual destas iterações você espera que vá convergir mais rapidamente?

Solução:

g1 e g2 são obtidas a partir de manipulações algébricas simples, já g3 correspondeao método de Newton. Para a iteração dada por gi(x) ser convergente, é necessárioque |g′i(x)| < 1 para todo x real. Mas,

|g′1(x)| = | cos(x)− 1| ≤ | cos(x)|+ |1| ≤ 2,

ao passo que|g′2(x)| = | cos(x)/2| ≤ 1/2.

Portanto, não podemos garantir a convergência de g1, mas g2 e g3 são convergen-tes. A terceira converge mais rapidamente que a segunda, porque a convergênciado método de Newton é quadrática, ao passo que a convergência da segundaiteração é apenas linear, pois g′2(x) 6= 0.

Questão 2 (2.0 pontos)Considere o sistema AX = b em que

A =

−4 1 04 −6 20 5 −8

e b =

1566

(a) Determine a decomposição PLU de A e resolva o sistema.

(b) Ache matrizes R e c tais que xk = Rxk + c é a iteração de Jacobi do sistema.

Page 18

Solução:

A decomposição PLU é dada por P = I,

L =

1 0 0−1 1 00 −1 1

e U =

−4 1 04 −5 20 0 −6

As soluções do sistema são

x = −21

4, y = −6, z = −9

2.

Decompondo A na forma

A =

−4 1 04 −6 20 5 −2

=

−4 0 00 −6 00 0 −2

+

0 1 04 0 20 5 0

temos que

R = −

0 −1/4 0−2/3 0 −1/30 −5/2 0

e c =

−15/4−1−3

Questão 3 (1.0 pontos)Qual o número n de partes em que é necessário dividir o intervalo [0, 10] para calculara área sob o grá�co de cos(x2) com erro inferior a 10−4, usando o método de Simpson?

Solução:

O erro no método de Simpson é dado por

− 1

180nh5f (4)(ξ) = − 1

180

(105

n4

)f (4)(ξ)

para algum ξ ∈ [0, 10]. Como

f 4(x) = 48 x2 sin(x2)+(16 x4 − 12

)cos(x2)

temos que|f 4(x)| =≤ 160388

para todo x ∈ [0, 10]. Logo, uma cota superior para o módulo do erro é dadaporque

| 1180

(105

n4

)5

f (4)(ξ)| ≤ 160388

180

(105

n4

).

Tomando,160388

180

(105

n4

)≤ 10−4,

obtemos n ≥ 943951.505 e, como n é inteiro, n ≥ 943952.

Page 19

Questão 4 (2.0 pontos)Considere os pontos (1,−3), (2, 1), (4, 51), (5, 109), (6, 197), (9, 701).(a) Use o método de diferenças divididas para encontrar o grau e o coe�ciente do

termo de maior grau do polinômio que interpola estes pontos.

(b) Use o método dos mínimos quadrados para achar o polinômio de grau 3 quemelhor se adapta a estes pontos.

Solução:

A tabela gerada pelo método de diferenças divididas é

1.0 −3.02.0 1.0 4.04.0 51.0 18.0 7.05.0 109.0 28.0 8.0 1.06.0 197.0 40.0 9.0 1.0 0.09.0 701.0 88.0 12.0 1.0 0.0 0.0

Como só precisamos dos quatro primeiros pontos para achar os coe�cientes dopolinômio interpolador, isto sign�ca que o polinômio é x3−3x−1, que tem grau 3e seu coe�ciente líder é 1.0. Como o polinômio calculado via mínimos quadradosminimiza a distância entre seu grá�co e os pontos dados e há um polinômio degrau três que passa por esses pontos, então o polinômio desejado é o mesmocalculado acima.

Questão 5 (2.0 pontos)Considere o problema de valores de contorno xy′′− 2y′ = 6, com y(0) = 0 e y(5) = 0.

(a) Determine o sistema linear nas variáveis y2, y3 e y4 obtido aplicando-se a esteproblema o método das diferenças �nitas com passo h = 1 e resolva-o.

(b) Calcule y′′′(1).

Solução:

Substituindo as aproximações

y′(xk) ≈yk+1 − yk−1

2he y′′(xk) ≈

yk+1 − 2yk + yk−1h2

na equação e levando em conta que h = 1, obtemos

(xk + 1)yk−1 − 2xkyk + (x− 1) ∗ yk−1 = 6.

Portanto, o sistema será

−2y1 = 6

−4y2 + y3 = 15

4y2 − 6y3 + 2y4 = 6

5y3 − 8y4 = 6.

Page 20

A matriz deste sistema é a mesma da segunda questão

A =

−4 1 04 −6 20 5 −2

cuja Para calcular y′′′(1), note que, derivando xy′′ − 2y′ = 6 obtemos

y′′ + xy′′′ − 2y′′ = 0,

donde

y′′′ =1

xy′′.

Quando x = 1, obtemos

y′′′(1) = y′′(1) = (6− 2y′(1)) ≈ 6− (y2 − y1) = 6− (−21

4+ 3) = −9

4.

Page 21

Primeira Prova�Turma EC2�UFRJ�2017.2

Justi�que cuidadosamente todas as suas respostas.

Questão 1 (2.5 pontos)Suponha que um computador C arredonda para duas casas decimais números escritosna notação padrão de ponto �utuante e considere as funções

f(x) = 1− sen(x) e g(x) =cos(x)2

1 + sen(x).

(a) Mostre que f(x) = g(x) e determine os valores obtidos se C for usado paracalcular f(1.5) e g(1.5).

(b) Sabendo-se que f(1.5) = g(1.5) = 0.002505013, determine o erro relativo corres-pondente a cada um dos cálculos executados em (a).

Bonus track: por que f(1.5) é menos preciso que g(1.5)?

Solução:

Obtemos f(x) substituindo

cos(x)2 = 1− sen(x)2 = (1− sen(x))(1 + sen(x))

em g(x) e cancelando 1 + sen(x) no numerador e denominador. Como

sen(1.5) = 0.9974949866 . . . e cos(1.5) = 0.0707372016,

então as representações destes números no computador C serão

sen(1.5) ≈ 1.00 e cos(1.5) ≈ 0.071.

Portanto,f(1.5) ≈ 1− 1 = 0,

ao passo que

0.712 = 0.005041 ≈ 0.005 e 1 + sen(1.5) ≈ 2

nos dão

g(1.5) ≈ 0.005

2= 0.0025.

Portanto os erros relativos correspondentes aos cálculos de f(1.5) e g(1.5) usandoo computador C serão, respectivamente,

|0.002505013− 0|0.002505013

= 1 e|0.002505013− 0.0025|

0.002505013= 0.002.

Portanto, g(1.5) tem um erro menor que f(1.5) e deve ser a forma preferida parao cálculo deste número. A razão pela qual f(1.5) produz um valor pior é que háuma �subtração catastró�ca� nesta função.

Page 22

� Duas casas decimais signi�ca que o computador representa os números na forma

0.a1a2 · 10m, com a1 obrigatoriamente não nulo. Como o computador C representa

os números com apenas 2 casas, é necessário arredondar cada vez que um cálculo é

realizado, e não apenas ao �nal. O erro relativo é de�nido como |xa − xe|/|xe|, em que

xa é o valor aproximado e xe o valor exato e não |xa − xe|/|xa|.

Questão 2 (2.5 pontos)Seja Pn(x) o polinômio de Taylor de f(x) = x ln(x) em x0 = 1.

(a) Calcule a expressão do erro absoluto |en| quando x > 1.

(b) Determine n tal que o erro absoluto cometido quando usamos Pn(1.01) comoaproximação de f(1.01) seja inferior a 10−11.

Solução:

Calculando algumas derivadas, vemos que

f ′(x) = ln(x) + 1

f ′′(x) = x−1

f ′′′(x) = −x−2

f (iv)(x) = 2x−3

f (v)(x) = −2 · 3 · x−4,donde podemos deduzir que

f (n)(x) = (−1)n(n− 2)! · x1−n

de modo que o módulo do erro absoluto desejado será

|en| =∣∣∣∣(n− 1)! · c−n

(n+ 1)!(x− 1)n+1

∣∣∣∣ = ∣∣∣∣ c−n

n(n+ 1)(x− 1)n+1

∣∣∣∣Como 1 < c < 1.01,

c−n ≤ 1.

Portanto, quando x = 1.01, temos a estimativa

|en| =∣∣∣∣ c−n

n(n+ 1)(x− 1)n+1

∣∣∣∣ ≤ 1

n(n+ 1)(0.01)n+1 =

1

n(n+ 1)102(n+1).

logo basta que1

n(n+ 1)102(n+1)< 10−11,

para que |en| ≤ 10−11. Tabelando os valores veri�camos que

1

4 · 5 · 1010= 0.5 · 10−11

quando n = 4 é o primeiro valor que dá menor que 10−11. Assim, P4(1.01) dá umerro absoluto inferior a 10−11.

� Várias pessoas calcularam a derivada errado!

Page 23

Questão 3 (2.5 pontos)Considere o problema de valor de contorno y′′−y′+xy = −4 com y(0) = 0 e y(5) = 0.

(a) Determine o sistema linear obtido quando o método das diferenças �nitas éaplicado a este problema com passo h = 1. (Não precisa resolvê-lo!)

(b) use pivoteamento parcial para calcular a decomposição PLU da matriz do sis-tema obtido em (a).

Solução:

Substituindo as aproximações

y′(xk) ≈yk+1 − yk−1

2he y′′(xk) ≈

yk+1 − 2yk + yk−1h2

e h = 1 na equação, obtemos

(2 xi − 4) yi + yi+1 + 3 yi−1 = −8.

Note que xi = x0 + hi = i. Escrevendo

n = (5− 1)/h = 4,

as condições de contorno serão

y0 = 0 e y5 = 0.

Quando k = 1,y2 − 2 y1 = −8,

pois y0 = 0 e x1 = 1. Por outro lado, quando k = 4, teremos x4 = 4 e

4 y4 + 3 y3 = −8.

Para k = 2 e k = 3 as equações serão, respectivamente,

y3 + 3 y1 = −8 e y4 + 2 y3 + 3 y2 = −8,

Obtemos, assim, o sistema

y2 − 2 y1 = −8,y3 + 3 y1 = −8,

y4 + 2 y3 + 3 y2 = −8,4 y4 + 3 y3 = −8

A matriz deste sistema é −2 1 0 03 0 1 00 3 2 10 0 3 4

Em seguida, aplicamos eliminação com pivoteamento parcial, para achar as ma-trizes P , L e U . O pivoteamento parcial requer que façamos a troca das duas

Page 24

primeiras linhas. Fazendo isto e eliminando a posição não nula da primeira coluna,obtemos as matrizes:

P =

0 1 0 01 0 0 00 0 1 00 0 0 1

, L =

1 0 0 0−2

31 0 0

0 0 1 00 0 0 1

, U =

3 0 1 00 1 2

30

0 3 2 10 0 3 4

.Mais uma vez o pivoteamento requer que troquemos a segunda e a terceira linhas,antes de fazer a eliminação, o que nos dá

P =

0 1 0 00 0 1 01 0 0 00 0 0 1

, L =

1 0 0 00 1 0 0−2

313

1 00 0 0 1

, U =

3 0 1 00 3 2 10 0 0 −1

3

0 0 3 4

Com uma última troca de linhas, chegamos às matrizes desejadas, que são:

P =

0 1 0 00 0 1 00 0 0 11 0 0 0

, L =

1 0 0 00 1 0 00 0 1 0−2

313

0 1

, U =

3 0 1 00 3 2 10 0 3 40 0 0 −1

3

Questão 4 (2.5 pontos)Segundo a lei de Lotka, a relação entre a quantidade x de publicações e a porcentagemy de autores (em um certo período) que publicaram x artigos é dada por y = cx−n.Os valores de c e n dependem da área de pesquisa que está sendo considerada. Atabela abaixo mostra a relação entre x e y para artigos importantes de física até 1900:

x 1 2 4 8y 60.79 15.20 3.80 0.95

(a) Use logaritmos em base 2 para reescrever a lei de Lotka como uma relação linear.

(b) Calcule estimativas de c e n, pelo método dos mínimos quadrados, usando osdados da tabela.

(c) Use os valores de c e n que você determinou para prever qual seria a porcentagemdo total de publicações representada pelos autores que publicaram 10 artigos.

Lembrete: log2(a) = log10(a)/ log10(2) ≈ 10 · log10(a)/3.

Page 25

Solução:

Aplicando logaritos em base 2 aos dois lados de y = cx−n, obtemos

log2(y) = log2(c)− n log2(x).

Para poder aplicar mínimos quadrados precisamos tabelar log2(y) contra log2(x):

log2(x) 0 1 2 3log2(y) 5.93 3.93 1.93 −0.07

Portanto,

V =

1 01 11 21 3

e b =

5.933.931.93−0.07

.Escrevendo ` = log2(c) e levando em conta que

V tV =

[4 66 14

]e

[11.87.59

],

precisamos apenas resolver o sistema

−6.0n+ 4.0l = 11.8

−14.0n+ 6.0l = 7.59,

cujas soluções sãon = 2.02 e l = 5.96.

Como,c = 25.96 = 62.3

a fórmula da lei de Lotka neste caso é

y = 62.3x−2.02.

Portanto, quando x = 10,

y = 62.3 · 10−2.02 = y = 0.59.

Os dados da tabela foram retirados do artigo original do Lotka:

A. J. Lotka, The frequency distribution of scienti�c productivity, Jour-nal of the Washington Academy of Sciences. 16 (1926), 317�324.

O valor exato para y em x = 10 é 0.61.

Page 26

Segunda Prova�Turma EC2�UFRJ�2017.2

Questão 1 (2.5 pontos)Considere a função f(x) = 2 sen(

√x)− x.

(a) Determine uma função g(x), diferente da obtida pelo método de Newton, cujoponto �xo é um zero de f(x).

(b) Sabendo-se que a iteração xk+1 = g(xk) tem um ponto �xo no intervalo [1.5, 3],prove que ela converge para este ponto no intervalo dado.

(c) Use g(x) para calcular o zero de f(x) em [1.5, 3] com erro inferior a 0.0001,começando de x0 = 1.5.

Solução:

Tomandog(x) = 2 sen(

√x),

temos que

g′(x) = −cos(√x)√

x;

de modo que

|g′(x)| = | cos(√x)|√

x≤ 1√

x.

Como 1/√x é decrescente quando x > 1, temos que

|g′(x)| ≤ 1√x<

1√1.5≈ 0.82,

o que garante a convergência de g(x) para o ponto �xo no intervalo dado. Ite-rando, obtemos

k xk |xk − xk−1|1 1.881439 0.3814392 1.960474 0.07903513 1.970957 0.0104834 1.972213 0.0012565 1.972361 0.0001486 1.972379 0.000018

� O maior problema nesta questão foi com a estimativa da cota superior para |g′(x)|.Muita gente testou apenas os extremos. Neste caso particular a função é decres-

cente, de modo que o máximo é atingido no extremo esquerdo do intervalo; mas, para

usar isto, você teria que veri�car que a função é decrescente, o que a vasta maioria não

fez. Além disso, não basta mostrar que |g′(x)| < 1 para todo x ∈ [1.5, 3]; é necessário

mostrar que existe um número real L < 1, para o qual |g′(x)| < L. Para entender

porque este detalhe é tão importante, leia a observação no alto da página 21 das notas

de aula.

Page 27

Questão 2 (1.5 pontos)Use interpolação entre os pontos x = 1, x = 2 e x = 4 para calcular log2(3).

Solução:

Comolog2(1) = 0, log2(2) = 1 e log2(4) = 2,

o polinômio interpolador será dado por

P (x) = −(x− 1)(x− 4)

2+ 2

(x− 1)(x− 2)

6.

Logo,

P (3) = −2 · (−1)2

+ 22 · 16

= 1 +2

3=

5

3.

Questão 3 (1.5 pontos)Tendo aplicado dez iterações do método da potência à matriz

A =

1 1 0−1 3 1−1 1 2

obtivemos o vetor v =

−0.401−0.816−0.415

.Use isto para calcular um dos autovalores de A.

Solução:

Como v é uma aproximação para um autovetor de A,

Av =

1 1 0−1 3 1−1 1 2

−0.401−0.816−0.415

=

−1.22−2.46−1.25

≈ λv

de modo que

λ ≈ −1.22−0.401

≈ 3.04.

Questão 4 (2.5 pontos)Use o método de Simpson para calcular a integral com erro inferior a 0.01∫ 1

0

x2 cos(x)dx.

Page 28

Solução:

Aplicando o método de Simpson com o intervalo de integração subdividido em2n partes iguais, obtemos∫ b

a

f(x)dx =h

3(y0 + y2n + 2

n−1∑i=1

y2i + 4n−1∑i=0

y2i+1)−(b− a)180

f (iv)(α)h4

para algum α ∈ (a, b), em que yi = f(a + ih) e h = (b − a)/2n. Começamosestimando o número de partes em que é necessário dividir o intervalo [0, 1] paraque o erro �que abaixo de 0.01. Como

f (iv)(x) = 8 x sin (x) +(x2 − 12

)cos (x)

temos que|f (iv)(x)| ≤ 8x+ x2 + 12 < 21,

para todo x ∈ [0, 1], donde o erro satisfaz

21

180h4 < 0.01.

Levando em conta que h = 1/n, obtemos

n4 >21

180 · 0.01= 11.667,

que nos dá n > 1.848. Portanto, o menor valor de n que podemos tomar é n = 2(note que estou usando n para representar a quantidade de bandas!) fazendo isto,obtemos a seguinte aproximação para a integral

1

6(y0 + y2 + 4y1) ≈ 0.055.

� Novamente o maior problema foi com a cota superior do erro. Desta vez o problema

foi mais sério, porque a função f (iv)(x) tem máximo igual 16.81 em x = 0.6569. Ográ�co da quarta derivada está ilustrado na �gura abaixo. Outro erro foi cometido por

algumas pessoas que calcularam a integral usando 0.5 como aproximação para cos(1) ≈0.5403023. Isto corresponde a um erro de mais de 0.04 para o cosseno, o que não é

compatível com obter um erro inferior a 0.01 para a integral.

Questão 5 (3.0 pontos)

Page 29

O método de Euler reverso consiste em aplicar a recorrência yk+1 = yk+hf(tk+1, yk+1)ao problema de valor inicial y = f(t, y) e y(0) = y0.

(a) Use o método de Euler reverso com h = 0.5 para calcular y(1), quando y(t) é asolução do problema de valor inicial y = cos(t) + 4y, com y(0) = 1.

(b) Calcule as fórmulas de Taylor com resto de ordem dois das funções y(t) e y(t)na vizinhança de tk.

(c) Calcule o erro de truncamento obtido quando o método de Euler reverso é apli-cado à equação autônoma y = f(y) e use-o para determinar a ordem destemétodo.

Solução:

(a) A recorrência do método de Euler reverso nos dá

yk+1 = yk + h(cos(tk+1) + 4yk+1)

donde(1− 4h)yk+1 = yk + h cos(tk+1).

Tomando h = 0.5, obtemos

yk+1 = −(yk + 0.5 cos(tk+1));

dondey1 = −1.44 e y2 = 1.17.

Assim, y(1) ≈ 1.17, que é impressionantemente ruim, porque y(1) = 67.367.Precisei de mil iterações para obter como aproximação 67.912 !!!

(b) As fórmulas de Taylor desejadas são dadas por

y(t) = y(tk) + y(tk)(t− tk) +y(α)

2(t− tk)2

e por

y(t) = y(tk) + y(tk)(t− tk) +...y (β)

2(t− tk)2,

em que α e β são números entre tk e t.

(c) O erro de truncamento do método de Euler reverso é dado por

Tk =y(tk + h)− y(tk)

h− f(y(tk + h)).

Levando em conta que, da equação diferencial, y(t) = f(y(tk + h)), podemosreescrever Tk na forma

Tk =y(tk + h)− y(tk)

h− y(tk + h).

Substituindo as fórmulas de Taylor e simpli�cando, obtemos

Tk = y(tk) +y(α)

2(α)h−

(y(tk) + y(tk)h+

...y (β)

2h2);

Page 30

donde

Tk = h

(y(α)

2− y(tk)−

...y (β)

2

)h,

de modo que o método de Euler reverso tem ordem um.

Page 31

Prova Final�Turma EC2�UFRJ�2017.2

Justi�que cuidadosamente todas as suas respostas.

Questão 1 (3.0 pontos)Considere o problema de valores de contorno xy′′− 2y′ = 6, com y(0) = 0 e y(5) = 0.

(a) Determine o sistema linear nas variáveis y2, y3 e y4 obtido aplicando-se a esteproblema o método das diferenças �nitas com passo h = 1.

(b) Calcule a decomposição PLU da matriz do sistema nas variáveis y2, y3 e y4obtido em (a) e resolva o sistema.

Solução:

Substituindo as aproximações

y′(xk) ≈yk+1 − yk−1

2he y′′(xk) ≈

yk+1 − 2yk + yk−1h2

na equação e levando em conta que h = 1, obtemos

(xk + 1)yk−1 − 2xkyk + (x− 1)yk−1 = 6.

Portanto, o sistema será

−2y1 = 6

−4y2 + y3 = 15

4y2 − 6y3 + 2y4 = 6

5y3 − 8y4 = 6.

Como, claramente, y1 = 3, basta considerar o sistema nas variáveis y2, y3 e y4. Amatriz deste sistema é

A =

−4 1 04 −6 20 5 −2

cuja decomposição PLU corresponde a

P =

1 0 00 1 00 0 1

, L =

1 0 0−1 1 00 −1 1

, U =

−4 1 00 −5 20 0 4

Resolvendo o sistema obtemos

y3 = −3

2, y2 = −

33

8, y4 =

27

4

Page 32

x 1 2 3 4y 4.29 12.80 39.21 119.59

Questão 2 (2.0 pontos)Sabe-se que f(x) é uma função da forma f(x) = c exp(bx) e que representam aproxi-mações de dos valores destas funções.

(a) Ache os valores de b e c para a função que melhor se ajusta aos dados da tabela.

(b) Calcule f(2.5) usando a função obtida em (a) e determine o erro relativo come-tido no cálculo de f(2.5) sabendo-se que o valor exato é 22.6145.

Solução:

Aplicando logaritmos em base 10 aos dois lados de y = bcx obtemos

ln(y) = bx+ ln(c).

Para poder aplicar mínimos quadrados precisamos tabelar ln(y) contra x:

x 1 2 3 4ln(y) 1.46 2.55 3.67 4.78

Portanto

V =

1 11 21 31 4

e b =

1.462.553.674.78

.Levando em conta que

V tV =

[4 1010 30

]e V tb =

[12.4636.69

]precisamos apenas resolver o sistema

4.0l + 10.0b = 12.46

10.0l + 30.0b = 36.69

em que l = ln(c), cujas soluções são

l = 0.345 e b = 1.108.

Como,c = exp(l) = 1.41

a aproximação para a fórmula de f(x) que obtivemos é

y = 1.41198 e(1.108 x).

Portanto, quando x = 2.5,y = 22.53.

Logo, o erro relativo cometido foi de

|22.53− 22.6145|22.6145

= 0.00358554.

Page 33

Questão 3 (1.5 pontos)As seguintes iterações foram propostas como maneiras de calcular a interseção dosgrá�cos das funções sen(x) e f(x) = −2x+ 2:

g1(x) = 2− sen(x)− xg2(x) = (2− sen(x))/2

g3(x) = (− sen(x) + x cos(x) + 2)/(2 + cos(x)).

(a) Explique como cada uma destas iterações foi obtida.

(b) Para quais destas iterações podemos garantir a convergência a partir de x = 1?

(c) Qual destas iterações você espera que vá convergir mais rapidamente?

Solução:

g1 e g2 são obtidas a partir de manipulações algébricas simples, já g3 correspondeao método de Newton. Para a iteração dada por gi(x) ser convergente, é necessárioque |g′i(x)| < 1 para todo x real. Mas,

|g′1(x)| = | cos(x)− 1| ≤ | cos(x)|+ |1| ≤ 2,

ao passo que|g′2(x)| = | cos(x)/2| ≤ 1/2.

Portanto, não podemos garantir a convergência de g1, mas g2 e g3 são convergen-tes. A terceira converge mais rapidamente que a segunda, porque a convergênciado método de Newton é quadrática, ao passo que a convergência da segundaiteração é apenas linear, pois g′2(x) 6= 0.

Questão 4 (1.5 pontos)Em quantas partes é necessário subdividir o intervalo [0, 1] para que a integral abaixopossa ser calculada usando o método de trapézio 10−5∫ 1

0

x3 exp(2x)dx.

Solução:

O erro no método do trapézio é dado por

(b− a)12

f ′′(α)h2

para algum α ∈ (a, b), em que h = (b− a)/2n. Como

f ′′(x) =(4 x3 + 12 x2 + 6 x

)e(2 x)

Page 34

é uma função crescente em [0, 1], temos que

|f ′′(x)| ≤ f ′′(1) = 162.559.

para todo x ∈ [0, 1], donde o erro satisfaz

162.559

12h2 < 10−5.

Levando em conta que h = 1/n, obtemos

n2 >162.559

12 · 10−5= 1354660,

que nos dá n > 1163.9. Portanto, seriam o menor valor de n que podemos tomaré n = 2. fazendo isto, obtemos a seguinte aproximação para a integral

1

12(y0 + y4 + 4(y1 + y3) + 2y2) ≈ 0.05.

Questão 5 (2.0 pontos)Considere o problema de valor inicial y = t2y3 com y(1) = 2.

(a) Determine a iteração obtida aplicando o método de Runge-Kutta de segundaordem a este problema com h = 0.5.

(b) Calcule o polinômio de Taylor de grau dois da solução y(t) do problema de valorinicial dado na vizinhança da origem.

Solução:

Aplicando a fórmula do método de Runge-Kutta de segunda ordem ao problemadado, obtemos a iteração

yk+1 = yk +h

2(t2ky

3k + t2k+1(yk + h(t2ky

3k)))).

Substituindo h = 0.5 e expandindo,

yk+1 = yk +1

4(t2ky

3k + (tk + 0.5)2(yk + 0.5(t2ky

3k)))).

De y = t2y3 obtemos

y = 2 t2 y (t) y (t) + 2 t y (t)2 = 2t4y(t)4 + 2ty(t)2;

de modo que y(0) = 8 e y(0) = 40. Logo, o polinômio de Taylor de grau dois dey(t) será P2(t) = 2 + 8t+ 40t2.

Page 35